Algebra learning

Polinomi, disuguaglianze, numeri complessi, ...
PG93
Messaggi: 29
Iscritto il: 17 nov 2017, 16:52

Re: Algebra learning

Messaggio da PG93 »

9.1
Testo nascosto:
Si verifica facilmente che $f(a,b,c)=\frac{a}{b+c+1} + \frac{b}{c+a+1} + \frac{c}{a+b+1} + (1-a)(1-b)(1-c) \leq 1$ definita per $a,b,c\in[0,1]$ è convessa in ognuna delle tre variabili. Perciò il massimo di $f$ è ottenuto per $a,b,c\in\{0;1\}$, cioè agli estremi del dominio di $f$. Verifichiamo che per queste otto terne si ha $f(a,b,c)=1$, e ciò permette di concludere :D
PS: suggerimenti per lo svolgimento sarebbero più che graditi ! :)
scambret
Messaggi: 734
Iscritto il: 23 mag 2012, 20:49
Località: Acquarica del Capo

Re: Algebra learning

Messaggio da scambret »

Giusta! Infatti il suggerimento c'è, solo che arriva due settimane dopo la pubblicazione ;)
PG93
Messaggi: 29
Iscritto il: 17 nov 2017, 16:52

Re: Algebra learning

Messaggio da PG93 »

Intendevo suggerimenti su come migliorare come ho scritto la soluzione. Infatti, generalmente le scrivo in un altra lingua, e non sono abituato a fare matematica "in italiano" :)
Avatar utente
Pit
Messaggi: 23
Iscritto il: 22 ago 2017, 10:22

Re: Algebra learning

Messaggio da Pit »

9.3.
Testo nascosto:

Chiamiamo $y_i=\frac{x_i}{2017}>0$ e $z_i=\frac{1}{y_i+1}>0$, da cui $y_i=\frac{1}{z_i}-1$. La condizione diventa $$\frac{1}{y_1+1}+\frac{1}{y_2+1}+...+\frac{1}{y_n+1}=z_1+z_2+...+z_n=1$$
mentre la tesi è equivalente a $$\sqrt[n]{y_1\cdots y_n}\geq n-1$$
Consideriamo la funzione $f(x)=\ln\left(\frac{1}{x}-1\right)$. Questa è convessa in $\left(0,\frac{1}{2}\right]$ e quindi se $0<z_i\leq\frac{1}{2}$ per ogni $i$ da 1 a $n$, per Jensen si ha $$\frac{f(z_1)+f(z_2)+...+f(x_n)}{n}\geq f\left(\frac{z_1+z_2+...+z_n}{n}\right)=f\left(\frac{1}{n}\right)\Rightarrow$$$$\frac{1}{n}\left[\ln\left(\frac{1}{z_1}-1\right)+\ln\left(\frac{1}{z_2}-1\right)+...+\ln\left(\frac{1}{z_n}-1\right)\right]\geq \ln(n-1)\Rightarrow$$$$\frac{1}{n}\left[\ln(y_1y_2\cdots y_n)\right]\geq \ln(n-1)\Rightarrow \ln\left(\sqrt[n]{y_1y_2\cdots y_n}\right)\geq\ln(n-1)\Rightarrow \sqrt[n]{y_1y_2\cdots y_n}\geq n-1$$
Se invece per almeno un $j$ si ha $z_j>\frac{1}{2}$, supponiamo $j=1$. Chiaramente questo è unico dato che se ce ne fosse almeno un altro si avrebbe $\sum\limits_{k=1}^n z_k>1$ che va contro la condizione.
Chiamiamo $a=\frac{z_1+z_2}{2}\leq\frac{1}{2}$. Vale la disuguaglianza $$\left(\frac{1}{a}-1\right)^2\leq \left(\frac{1}{z_1}-1\right)\left(\frac{1}{z_2}-1\right)=y_1y_2$$ (se non ho sbagliato i calcoli) questa dovrebbe essere equivalente a $(z_1-z_2)^2(1-z_1-z_2)\geq 0$ . Per Jensen sulla n-upla $(a,a,z_3,...,z_n)$ si ottiene similmente a prima $$n-1\leq \sqrt[n]{\left(\frac{1}{a}-1\right)^2y_3y_4\cdots y_n}\leq\sqrt[n]{y_1y_2\cdots y_n}$$ che è la tesi.
Nessuno :?:
scambret
Messaggi: 734
Iscritto il: 23 mag 2012, 20:49
Località: Acquarica del Capo

Re: Algebra learning

Messaggio da scambret »

Pronti per il WC? Nuova sessione di A!

Hint sui problemi 9
Testo nascosto:
9.1. La funzione è convessa, dunque il massimo sta sicuramente sul bordo. È anche simmetrica, dunque bastano 4 prove.

9.2. Cambiamo le variabili in $x_i = 2 - a_i$ e supponiamo che la tesi non vale. Allora tutti gli $x_i$ sono positivi, $\displaystyle \sum x_i^2 > n^2$ e $\displaystyle \sum x_i \leq n$. Da qui si ottiene l’assurdo.

9.3. Ponendo $t_i = 2017/(2017+x_i)$, dunque $\displaystyle \sum t_i = 1$. Ora AM-GM o smoothing.
10.1. Trovare tutte le funzioni biettive $f, g, h: \mathbb{N} \to \mathbb{N}$ tali che per ogni $n \in \mathbb{N}$ vale

$$[f(n)]^3+[g(n)]^3+[h(n)]^3 = 3ng(n)h(n)$$

10.2. Trovare tutte le funzioni $f: \mathbb{N} \to \mathbb{N}$ per cui si ha che

$$f(n+1)>f(f(n))$$

10.3. Trovare tutte le funzioni $f: \mathbb{Z} \to \mathbb{N}$ per cui abbiamo

$$6f(k+3)-3f(k+2)-2f(k+1)-f(k)=0$$
Avatar utente
Sirio
Messaggi: 317
Iscritto il: 08 set 2016, 22:01

Re: Algebra learning

Messaggio da Sirio »

10.1
Testo nascosto:
Trattiamo prima il caso $n=0$ e poi quello $n>0$.

Sostituendo $n=0$ nell'equazione del testo otteniamo:
$\left(f(0)\right)^3+\left(g(0)\right)^3+\left(h(0)\right)^3=0$
Da cui, ricordando che $f(0)$ e cicliche sono naturali, quindi sono maggiori o uguali a zero, quindi la somma dei loro cubi è maggiore o uguale a zero, otteniamo che affinché quest'ultima somma sia nulla è necessario $f(0)=g(0)=h(0)=0$ $(1)$.

Supponiamo ora $n>0$. Per la $(1)$ e per l'ipotesi che le tre funzioni sono biunivoche e con immagine naturale abbiamo $f(n)>0$ e cicliche.
Dividiamo per tre i due membri dell'equazione del testo ed estraiamone quindi la radice cubica:
$\displaystyle \sqrt[3]{\dfrac{\left(f(n)\right)^3+\left(g(n)\right)^3+\left(h(n)\right)^3}3}=\sqrt[3]{n·g(n)·h(n)}\;\;\;\forall n\in\mathbb N*$ $(2)$
Applicando ora CM-GM otteniamo:
$\displaystyle \sqrt[3]{\dfrac{\left(f(n)\right)^3+\left(g(n)\right)^3+\left(h(n)\right)^3}3}\geq\sqrt[3]{f(n)·g(n)·h(n)}\;\;\;\forall n\in\mathbb N*$
Da cui, per la $(2)$:
$\sqrt[3]{n·g(n)·h(n)}\geq\sqrt[3]{f(n)·g(n)·h(n)}\;\;\;\forall n\in\mathbb N*$
Da cui:
$n≥f(n)\;\;\;\forall n\in\mathbb N*$
Ecco, qui vorrei sapere da voi se posso semplicemente dire che visto che è biunivoca allora ho $f(n)=n\;\;\;\forall n\in\mathbb N*$ oppure se ci devo spendere due parole in più...
Tornando alla $(2)$, sostituendo $f(n)=n$ otteniamo un'uguaglianza tra media cubica e media geometrica, da cui segue $g(n)=h(n)=n\;\;\;\forall n\in\mathbb N*$.

Quindi, ricordando anche la $(1)$, abbiamo che $f,g,h$ sono tutte e tre l'identità.
$T=\sqrt{\dfrac l g 12\pi}$
scambret
Messaggi: 734
Iscritto il: 23 mag 2012, 20:49
Località: Acquarica del Capo

Re: Algebra learning

Messaggio da scambret »

Beh io considererei il minimo $m>0$ tale che $f(m)<m$ e concludi.

Pensa ora per $\mathbb{Q}$ e $\mathbb{R}$ cosa succede.
scambret
Messaggi: 734
Iscritto il: 23 mag 2012, 20:49
Località: Acquarica del Capo

Re: Algebra learning

Messaggio da scambret »

Hint sui problemi 10
Testo nascosto:
10.1. Per AM-GM si dimostra che $f(n) \leq n$.

10.2. Notiamo che $f(k) \geq k$. Ora supponiamo esista l’elemento più piccolo che non rispetta l’uguaglianza.

10.3. Questa sembra una ricorsione, in realtà non lo è. Ma l’immagine è in $\mathbb{N}$, dunque esiste un minimo dell’immagine $a$. Posto $f(x)=a$, allora $k+3=a$. Da qui si conclude


11.1. Siano $a_1, \cdots, a_n$ reali non negativi. Dimostrare che

$$\left( \frac{1}{a_1} + a_2 + 2 \right) \left( \frac{1}{a_2} + a_3 + 2 \right) \cdots \left( \frac{1}{a_n} + a_1 + 2 \right) \leq \frac{(a_1+1)^2(a_2+1)^2 \cdots (a_n+1)^2}{a_1a_2 \cdots a_n}$$

11.2. Siano $a_1, \cdots, a_n$ reali non negativi. Dimostrare che

$$\prod_{cyc} (a_i^3+a_{i+1}+1) \geq \prod_{cyc} (a_ia_{i+1}^2+a_{i+1}+1)$$

11.3. $a, b, c$ lati del triangolo con $2A^2 \geq P$ ($A$ è l’area e $P$ il perimetro), dimostrare che

$$(a+b)(b+c)(c+a) \leq a^2b^2c^2$$
scambret
Messaggi: 734
Iscritto il: 23 mag 2012, 20:49
Località: Acquarica del Capo

Re: Algebra learning

Messaggio da scambret »

Hint sui problemi 11
Testo nascosto:
Quando avete prodotti dappertutto, potrebbe essere utile usare il seguente lemma RR: $a$ e $b$ vettori ordinati in maniera simile, allora

$$(a_1 + b_1) \cdots (a_n+b_n) \leq (a_1 + b_{\sigma(1)}) \cdots (a_n + b_{\sigma(n)})$$

La dimostrazione è simile al riarrangiamento.

Un corollario banale è che se $f$ è una funzione crescente e positiva, allora vale

$$(a_1 + f(a_1)) \cdots (a_n+f(a_n)) \leq (a_1 + f(a_{\sigma(1)})) \cdots (a_n + f(a_{\sigma(n)}))$$

11.1. Notiamo che $\displaystyle \frac{(a_1+1)^2}{a_i} = a_1 + 2 + \frac{1}{a_1}$. Notando che la funzione $2+1/x$ è decrescente si può usare RR al contrario.

11.2. Al LHS abbiamo $a_1^3+a_2+1$. Se consideriamo la funzione $f(x)=x^3+1$ che è crescente, abbiamo che LHS si può minorizzare con $\displaystyle \prod_{cyc} (a_1^3+a_1+1)$. Ora se cambiamo il RHS con $\displaystyle a_{2}^2(a_1 + \frac{1}{a_2} + \frac{1}{a_2^2})$ e il CHS con $\displaystyle a_{1}^2(a_1 + \frac{1}{a_1} + \frac{1}{a_1^2})$, basta applicare il RR a $f(x)=1/x+1/x^2$ che è decrescente.

11.3. La condizione sull’area e sul perimetro è davvero bruttissima. Calcolando tutto e con la Ravi, ci rimane $xyz \geq 1$ e la disuguaglianza iniziale come $\displaystyle \prod_{cyc} (x+2y+z) \leq \prod_{cyc} (x^2+2xy+y^2)$. Possiamo dire che $LHS \leq xyz LHS$, dividendo per $x^2y^2z^2$ entrambi i membri e ponendo $l=x/y$, $m=y/z$ e $n=z/x$ si ottiene $\displaystyle \prod_{cyc} (l+2+1/m) \leq \prod_{cyc} (l+2+1/l)$ e con $f(x)=2+1/x$ è davvero finita.
12.1. Determinare per quali valori reali di $a$ esiste una funzione razionale $f(x)$ che soddisfa

$$f(x^2)=[f(x)]^2-a$$

(Si ricorda che una funzione razionale è il rapporto tra due polinomi qualunque).

12.2. Trovare tutte le 4-uple di polinomi $(P, Q, R, S)$ tali che, se $x, y, z, t$ sono naturali con $xy-zt=1$, allora

$$P(x)Q(y)-R(z)S(t) = 1$$

12.3. Se $P$ e $Q$ sono dei polinomi monici tali che per ogni $x \in \mathbb{R}$ vale $P(P(x))=Q(Q(x))$, allora dimostrare che $P=Q$.
Avatar utente
Sirio
Messaggi: 317
Iscritto il: 08 set 2016, 22:01

Re: Algebra learning

Messaggio da Sirio »

Nel 12.1, escludiamo le $f$ costanti, giusto?
$T=\sqrt{\dfrac l g 12\pi}$
scambret
Messaggi: 734
Iscritto il: 23 mag 2012, 20:49
Località: Acquarica del Capo

Re: Algebra learning

Messaggio da scambret »

Escludeteli pure!
TheRoS
Messaggi: 30
Iscritto il: 25 feb 2018, 13:05

Re: Algebra learning

Messaggio da TheRoS »

Provo il 12.3
Per prima cosa notiamo che $P(P(x))=Q(Q(x))$ perché assumono lo stesso risultato per infiniti valori iniziali. Inoltre possiamo constatare che $deg(P(x))=deg(Q(x))$: il grado di $P(P(x))$ è dato dal grado di $P(x)$ (che chiameremo $d_p$) moltiplicato per sé stesso (l'esponete di grado maggiore di $P(x)$ viene elevato al grado di $P(x)$) quindi è $d_p^2$. Lo stesso discorso vale per il grado di $Q(Q(x))$ che, chiamato $d_q$ il grado di $Q(x)$, è uguale a $d_q^2$. Da qua si ottiene $d_p^2=d_q^2\Longrightarrow d_p=d_q$: tale grado lo chiameremo d'ora in avanti $n$.
A questo punto scriviamo $P(x)=x^n+\lambda_{n-1}x^{n-1}+\dots+\lambda_0$ e $Q(x)=x^n+c_{n-1}x^{n-1}+\dots+c_0$, la tesi si ripropone quindi nel dimostrare che $\lambda_i=c_i$ per ogni $i\in{0,\dots,n-1}$.
$P(P(x))=Q(Q(x))\Longrightarrow P(x)^n+\lambda_{n-1}P(x)^{n-1}+\dots+\lambda_0=Q(x)^n+c_{n-1}Q(x)^{n-1}+\dots+c_0$. Prendiamo in considerazione $P(x)^n$, tale termine ha come grado $n^2$, mentre $P(x)^{n-1}$ ha come grado $n(n-1)$; come si può notare c'è uno stacco di $n$ tra i due termini: ciò significa che gli $n$ coefficienti di grado compreso $n^2$ e $n(n-1)+1$ (estremi inclusi), sono gli stessi del polinomio $P(P(x))$, chiaramente il discorso è analogo per $Q(x)$. Chiamiamo $k_n,\dots,k_1$ nell'ordine i coefficienti di $P(P(x))$ di grado da $n^2$ a $n(n-1)+1$. Il coefficiente $k_n$ sappiamo che è uguale a 1: infatti tale termine si può formare solamente moltiplicando tra loro i termini di grado $n$ degli $n$ polinomi $P(x)$ che formano $P(x)^n$ (e $Q(x)$), siccome per ipotesi si tratta di polinomi monici, sarà sicuramente 1. Per quanto riguarda $k_{n-1}$ si crea utilizzando solamente termini di grado $n$ ed $n-1$, nell'assurdità in cui si utilizzasse un termine di grado inferiore come $n-2$, anche se cercassi di compensare utilizzando esclusivamente termini di grado $n$ otterrei $n^2-2<n^2-1$, inoltre notiamo che di grado $n-1$ compare esattamente una volta in ogni prodotto per la formazione di un termine di grado $n-1$, basta che compaia due volte che non si può più compensare neanche utilizzando termini di grado $n$ (otterrei infatti $n^2-2<n^2-1$). Quindi $k_{n-1}$ è costituito da un'espressione con una sola variabile ($\lambda_{n-1}$) ovvero un polinomio di grado 1 valutato in $\lambda_{n-1}$ ( tale polinomio lo chiameremo $A_{n-1}(x)$), però $k_{n-1}$ si può scrivere anche utilizzando la variabile $c_{n-1}$ e anche questa è un'espressione in cui $c_{n-1}$ compare solo con grado 1, in particolare tale espressione si può scrivere come $A_{n-1}(c_{n-1})$ (le due espressioni, per costruzione del prodotto, sono infatti simmetriche). Quindi $A_{n-1}(\lambda_{n-1})=A_{n-1}(c_{n-1})\Longrightarrow \lambda_{n-1}=c_{n-1}$.
Più in generale il procedimento può essere iterato con un'induzione anche sugli altri $k_i$. Prima di fare ciò generalizziamo però alcune osservazioni.

Osservazione 1. Ogni coefficiente $k_i$ può essere formato solamente con la somma di termini che sono prodotti di monomi di grado maggiore o uguale a $i$. Basta un termine di grado $i-1$ che non posso più compensare il prodotto neanche utilizzando tutti gli altri termini con grado $n$. Si otterrebbe infatti come risultato $n(n-1)+i-1<n(n-1)+i$.

Osservazione 2. Un termine di grado $n(n-1)+i$ può essere formato solamente con prodotti di monomi in cui un termine di grado $i$ compare al più una volta. Basta infatti che ne compaiono 2 che non riesco più a compensare la somma neanche con termini di grado $n$ perché otterrei $n(n-2)+2i<n(n-1)+i$.

A questo punto possiamo applicare l'induzione. Vogliamo dimostrare che se $\lambda_j=c_j$ allora $\lambda_{j-1}=c_{j-1}$.
Il passo base ce l'abbiamo perché $\lambda_{n-1}=c_{n-1}$. Per quanto riguarda l'ipotesi induttiva sappiamo che se $\lambda_j=c_j$ allora tutti per tutti gli $i$ compresi tra $n$ e $j$ $\lambda_i=c_i$. Inoltre per le osservazioni fatte precedentemente si ricava che $k_{j-1}$ è un polinomio che presenta una sola variabile (che può essere o $\lambda_{j-1}$ o $c_{j-1}$) ed è di grado 1 per l'osservazione 2; quindi chiamando $A_{j-1}(x)$ il polinomio, si ottiene che $A_{j-1}(\lambda_{j-1})=A_j(c_{j-1})\Longrightarrow\lambda_{j-1}=c_{j-1}$.
Tale induzione prosegue fino a $k_1$, quindi ci manca ancora da dimostrare che $\lambda_0=c_0$.
Per questo di si può ragionare molto similmente tirando fuori un coefficiente $k_0$ che regge un termine di grado $n(n-1)$. Le espressioni sono sempre simmetriche da entrambe le parti per le variabili $\lambda_0$ e $c_0$ perché abbiamo conquistato tutti gli altri coefficienti, di conseguenza $\lambda_0=c_0$.
Mi scuso per eventuali typo.
scambret
Messaggi: 734
Iscritto il: 23 mag 2012, 20:49
Località: Acquarica del Capo

Re: Algebra learning

Messaggio da scambret »

È stata una sessione più difficile?

Hint sui problemi 12
Testo nascosto:
12.1. Scriviamo $f=P/Q$ con $Q$ monico. Allora $P$ è monico, $Q(x^2)=[(Q(x)]^2$ e dunque $Q(x)=x^n$. Da qui basta comparare i coefficienti.

12.2. Scambiando $x$ e $y$ si ottiene che $Q=cP$ e $S=dR$. Ora fissato $n$ e posto $y=n/x$ bisogna dimostrare che $P=x^n$ e $R=x^m$. Adesso è quasi finita.

12.3. Supponiamo $R=P-Q$ e sia $k>0$ il grado di $R$. Scrivendo l’uguaglianza data come $PP-QQ=QP-QQ+RP$ e confrontando il grado si ottiene una contraddizione. Se $R \equiv c$, allora sostituendo si ottiene che il polinomio $Q$ rispetta $Q(y+c) \equiv Q(y)-c$ e dunque $c=0$.
13.1. $a_1, \cdots, a_n$ sono reali non negativi con somma 1. Dimostrare che

$$a_1a_2 + a_2a_3 + \cdots + a_{n-1}a_n \leq \frac{1}{4}$$

13.2. Siano $a, b, c >0$ tali che $abc=8$. Dimostrare che

$$\sum_{cyc} \frac{a^2}{\sqrt{(1+a^3)(1+b^3)}} \geq \frac{4}{3}$$

13.3. Siano $a, b, c, d$ reali positivi tali che

$$\sum_{cyc} \frac{1}{1+a^4} = 1$$

Dimostrare che $abcd \geq 3$.
gup
Messaggi: 14
Iscritto il: 25 ago 2017, 22:49

Re: Algebra learning

Messaggio da gup »

$ $13.3 Ci provo, ma non sono sicuro. $
$Uso Cauchy-Schwarz su $\displaystyle {\left( \frac {1}{\sqrt{1+a^4}} \right)}$ e cicliche e su ($\sqrt{1+a^4}$ e cicliche).
Si ottiene $$\displaystyle {\sum_{cyc}\frac{1}{1+a^4}} \displaystyle{\sum_{cyc} 1+a^4}\geq (4)^2=16$$
Quindi $$\displaystyle{\sum_{cyc} 1+a^4}\geq16 \Rightarrow a^4+b^4+c^4+d^4\geq 12\Rightarrow \frac{a^4+b^4+c^4+d^4}{4}\geq 3 $$
Per AM-GM si ha $\frac{a^4+b^4+c^4+d^4}{4}\geq (a^4b^4c^4d^4)^{\frac{1}{4}}=abcd$. Quindi $abcd$ è il minimo di $\frac{a^4+b^4+c^4+d^4}{4}$ e questo minimo si ottiene se e solo se $a=b=c=d$. Se $abcd <3$, segue che per $a=b=c=d$ $$\frac{a^4+b^4+c^4+d^4}{4}<3$$ che è assurdo, da cui la tesi.$ $
Ultima modifica di gup il 04 mar 2018, 22:03, modificato 1 volta in totale.
scambret
Messaggi: 734
Iscritto il: 23 mag 2012, 20:49
Località: Acquarica del Capo

Re: Algebra learning

Messaggio da scambret »

Fino a $a^4+b^4+c^4+d^4 \geq 12$ ok.
Anche questo è giusto: $a^4+b^4+c^4+d^4 \geq 4abcd$

Ma supponi che $(a,b,c,d)=(3,0,0,0)$. Le due disuguaglianze sono verificate, ma non $abcd \geq 3$

In generale, se hai $X \geq Z$ e $Y \geq Z$, è difficile concludere $X \geq Y$
Rispondi